Products of ideals of K[x1, x2, x3, x4]

  • MHB
  • Thread starter Math Amateur
  • Start date
In summary: They are not the same. Why? Because $b \in B$ and $b \notin A$. In other words, $b \in B \setminus A$, and since $B \setminus A \neq \emptyset$, $A \neq B$.Similarly, the set $A = \{x_1x_3+x_2x_4\}$ and the set $B = \{fg: f \in I, g \in J\}$ cannot be the same set, because:$x_1x_3+x_2x_4 \in A$$x_1x_3+x_2x_4 \notin B$Therefore, $B
  • #1
Math Amateur
Gold Member
MHB
3,990
48
Products of ideals of K[x1, x2, x3, x3]

I am reading R.Y. Sharpe: Steps in Commutative Algebra. In chapter 2 on Ideals, on page 28 we find Exercise 2.27 which reads as follows: (see attachment)

------------------------------------------------------------------------------
2.27 Exercise: Let \(\displaystyle K\) be a field, and let \(\displaystyle R = K[x_1, x_2, x_3, x_4] \), the ring of polynomials over K in indeterminates x_1, x_2, x_3, x_4.

Set \(\displaystyle I = Rx_1 + Rx_2 \) and \(\displaystyle J = Rx_3 + Rx_4 \)

Show that \(\displaystyle IJ \ne \{fg: \ f \in I, g \in J \} \)
------------------------------------------------------------------------------

My problem is I seemed to have ended up showing that \(\displaystyle IJ = \{fg: \ f \in I, g \in J \} \) ... so obviously something is wrong with my working ...

Can someone please explain my error(s)?

My working is as follows:

--------------------------------------------------------------------------
\(\displaystyle Rx_1 = \{ fx_1 \ | \ f \in R \} \)

and Rx_2, Rx_3, Rx_4 are defined similarly.\(\displaystyle I = Rx_1 + Rx_2 \)

\(\displaystyle = \{ h+k \ | \ h \in Rx_1 , k \in Rx_2 \} \)

\(\displaystyle = \{ fx_1 + gx_2 \ | \ f, g \in R \} \)

and similarly

\(\displaystyle J = \{ hx_1 + kx_2 \ | \ h, k \in R \} \)

Then \(\displaystyle IJ = \) set of all finite sums of elements of the form \(\displaystyle lm \) with \(\displaystyle l \in I, m \in J \)

\(\displaystyle = \{ {\sum}_{i=1}^{n} l_im_i \ | \ n \in \mathbb{N}, l_i \in I, m_i \in J \} \)

\(\displaystyle = \{ {\sum}_{i=1}^{n} (f_ix_1 + g_ix_2)(h_ix_3 + k_ix_4) \ | \ f_i, g_i, h_i, k_i \in R \} \)

\(\displaystyle = \{ {\sum}_{i=1}^{n} f_ih_ix_1x_3 + f_ik_ix_1x_4 + g_ih_ix_2x_3 + g_ik_ix_2x_4 \ | \ f_i, g_i, h_i, k_i \in R \} \)

\(\displaystyle = \{ {\sum}_{i=1}^{n} l_ix_1x_3 + m_ix_1x_4 + p _ix_2x_3 + q_ix_2x_4 \ | \ l_i, m_i, p_i, q_i \in R \} \)

\(\displaystyle = lx_1x_3 + mx_1x_4 + px_2x_3 + x_2x_4 \ | \ l, m, p, q \in R \}\)

since we can put \(\displaystyle l_1 + l_2 + ... \ ... l_n = l \) and similarly with \(\displaystyle m, p, q \)Now consider the set \(\displaystyle \{ fg: \ f\in I, g \in J \} \)

\(\displaystyle \{ fg: \ f\in I, g \in J \} \)

\(\displaystyle = \{ (l_1x_1 + m_1x_2)(p_1x_3 + q_1x_4) \ | \ l_1, m_1, p_1, q_1 \in R \} \)

\(\displaystyle = \{ l_1p_1x_1x_3 + l_1q_1x_1x_4 + m_1p_1x_2x_3 + m_1q_1x_2x_4 \ | \ l_1, m_1, p_1, q_1 \in R \} \)

\(\displaystyle = \{ lx_1x_3 + mx_1x_4 + px_2x_3 + qx_2x_4 \ | \ l, m, p, q \in R \} \)BUT then \(\displaystyle IJ = \{fg: \ f \in I, g \in J \} \) ?

Can someone please explain my error(s)

Peter
 
Last edited:
Physics news on Phys.org
  • #2
Well clearly we have:

$\{fg: f \in I, g \in J\} \subseteq IJ$, so there must be something in $IJ$ that is NOT in $\{fg: f \in I, g \in J\}$.

Consider $x_1x_3 + x_2x_4$.

Clearly, we have:

$x_1 = x_1(1) + x_2(0) \in I$
$x_3 = x_3(1) + x_4(0) \in J$

$x_2 = x_1(0) + x_2(1) \in I$
$x_4 = x_3(1) + x_4(0) \in J$

so $x_1x_3,x_2x_4 \in IJ$, and thus so is their sum.

Now if:

$x_1x_3 + x_2x_4 = (x_1f + x_2g)(x_3h + x_4k),\ f,g,h,k \in K[x_1,x_2,x_3,x_4]$

$x_1x_3 + x_2x_4 = x_1x_3(fh) + x_1x_4(fk) + x_2x_3(gh) + x_2x_4(gk)$

so:

$fh = gk = 1 \implies f,g,h,k \neq 0$
$fk = gh = 0 \implies f = 0$ or $k = 0$, and $g = 0$ or $h = 0$.

In general, we only get "some" $R$-linear combinations of $x_1x_3,x_1x_4,x_2x_3,x_2x_4$ in the set $\{fg: f\in I,g \in J\}$ whereas in $IJ$ we get ALL of them. In other words showing that every element of $\{fg: f\in I,g \in J\}$ is of the same form as a "typical" element of $IJ$ only shows containment one way, you have to show that ANY such element of $IJ$ can be obtained that way, and it suffices to exhibit just ONE that cannot.
 
  • #3
Deveno said:
Well clearly we have:

$\{fg: f \in I, g \in J\} \subseteq IJ$, so there must be something in $IJ$ that is NOT in $\{fg: f \in I, g \in J\}$.

Consider $x_1x_3 + x_2x_4$.

Clearly, we have:

$x_1 = x_1(1) + x_2(0) \in I$
$x_3 = x_3(1) + x_4(0) \in J$

$x_2 = x_1(0) + x_2(1) \in I$
$x_4 = x_3(1) + x_4(0) \in J$

so $x_1x_3,x_2x_4 \in IJ$, and thus so is their sum.

Now if:

$x_1x_3 + x_2x_4 = (x_1f + x_2g)(x_3h + x_4k),\ f,g,h,k \in K[x_1,x_2,x_3,x_4]$

$x_1x_3 + x_2x_4 = x_1x_3(fh) + x_1x_4(fk) + x_2x_3(gh) + x_2x_4(gk)$

so:

$fh = gk = 1 \implies f,g,h,k \neq 0$
$fk = gh = 0 \implies f = 0$ or $k = 0$, and $g = 0$ or $h = 0$.

In general, we only get "some" $R$-linear combinations of $x_1x_3,x_1x_4,x_2x_3,x_2x_4$ in the set $\{fg: f\in I,g \in J\}$ whereas in $IJ$ we get ALL of them. In other words showing that every element of $\{fg: f\in I,g \in J\}$ is of the same form as a "typical" element of $IJ$ only shows containment one way, you have to show that ANY such element of $IJ$ can be obtained that way, and it suffices to exhibit just ONE that cannot.

Thanks so much for the help, Deveno ... ... but just a clarification question ...

It is clear from the first part of your analysis that

\(\displaystyle x_1x_3 + x_2x_4 \in IJ \)

Now presumably (?), what we want to do next is to show that

\(\displaystyle x_1x_3 + x_2x_4 \notin \{ fg \ | \ f \in I, g \in J \} \) ... ... (1)

Now, presumably, the steps after you write "Now if:" do exactly that - but I cannot see how what you have done shows (1) above.

Can you please explain the logic ...

Thanks again,

Peter
 
  • #4
If a set $A$ has an element the set $B$ does not, then the two sets cannot possibly be the SAME set.

For example, the set $A = \{a\}$ and the set $B = \{a,b\}$ are not the same (provided $a \neq b$), since $b \in B$, but $b \not \in A$.

In general, the problem with the set:

$K = \{xy: x \in I, y\in J\}$ for two ideals $I,J$ of a ring $R$ is that it is usually not closed under addition, so $(K,+)$ is not a subgroup of $(R,+)$.

The exercise you have been given to prove is typically the "standard example" for demonstrating this, compare exercise 3 (b) here:

http://math.berkeley.edu/~daffyd/113s11/hw6sol.pdf
 
  • #5
Deveno said:
If a set $A$ has an element the set $B$ does not, then the two sets cannot possibly be the SAME set.

For example, the set $A = \{a\}$ and the set $B = \{a,b\}$ are not the same (provided $a \neq b$), since $b \in B$, but $b \not \in A$.

In general, the problem with the set:

$K = \{xy: x \in I, y\in J\}$ for two ideals $I,J$ of a ring $R$ is that it is usually not closed under addition, so $(K,+)$ is not a subgroup of $(R,+)$.

The exercise you have been given to prove is typically the "standard example" for demonstrating this, compare exercise 3 (b) here:

http://math.berkeley.edu/~daffyd/113s11/hw6sol.pdf

Thanks Deveno ... I believe I understand what you have said in this post ... but the mechanics in the previous post still escape me ... I think that I did not explain my difficulties clearly enough ... so I will try to explain myself more explicitly and carefully ...So ... I can see that

\(\displaystyle x_1x_3 + x_2x_4 \in IJ \)

Now we want to show that

\(\displaystyle x_1x_3 + x_2x_4 \notin \{ fg \ | \ f \in I, g \in J \} \)So in doing this you begin:

"Now if:

$x_1x_3 + x_2x_4 = (x_1f + x_2g)(x_3h + x_4k),\ f,g,h,k \in K[x_1,x_2,x_3,x_4]$"

Here you are just expanding out the fact that

\(\displaystyle x_1x_3 + x_2x_4 \in IJ \) given that

\(\displaystyle I = Rx_1 + Rx_2 \) and \(\displaystyle J = Rx_3 + Rx_4 \)
Then you write:

"$x_1x_3 + x_2x_4 = x_1x_3(fh) + x_1x_4(fk) + x_2x_3(gh) + x_2x_4(gk)$" ... ... (1)

This is just multiplying out ... ...Then you write:

"so:

$fh = gk = 1 \implies f,g,h,k \neq 0$
$fk = gh = 0 \implies f = 0$ or $k = 0$, and $g = 0$ or $h = 0$."

These are just conditions on \(\displaystyle f, g, h, k \) that follow from equation (1) above ...
BUT ... how/why exactly do the above steps show that

\(\displaystyle IJ \ne \{fg: \ f \in I, g \in J \} \)

Sorry to be so pedestrian with this ... but hoping you can help ...

Peter
 
Last edited:
  • #6
$f$ (for example) cannot be 0 AND non-zero at the same time. Therefore, such an $f$ DOES NOT EXIST.

If we accept that $f \neq 0$, then $k$ HAS to be 0, which is again a contradiction. We cannot escape this...we have 4 non-zero polynomials, yet 2 of them (it doesn't really matter WHICH two) have to be 0.

So the existence of the $f,g,h,k$ we have to have if $x_1x_3 + x_2x_4$ is to be in the product set is impossible, which means that there is NO such factorization of $x_1x_3 + x_2x_4$, which means it is not in the set: $\{fg: f \in I,g \in J\}$.
 
  • #7
Deveno said:
$f$ (for example) cannot be 0 AND non-zero at the same time. Therefore, such an $f$ DOES NOT EXIST.

If we accept that $f \neq 0$, then $k$ HAS to be 0, which is again a contradiction. We cannot escape this...we have 4 non-zero polynomials, yet 2 of them (it doesn't really matter WHICH two) have to be 0.

So the existence of the $f,g,h,k$ we have to have if $x_1x_3 + x_2x_4$ is to be in the product set is impossible, which means that there is NO such factorization of $x_1x_3 + x_2x_4$, which means it is not in the set: $\{fg: f \in I,g \in J\}$.

My apologies Deveno .. I do see the point regarding f, g, h, and k but that leaves me more perplexed ...

I will try to explain clearly ...

Well ... the fact that \(\displaystyle x_1x_3 + x_2x_4 \in IJ \) and that \(\displaystyle I = Rx_1 + Rx_2 \) and \(\displaystyle J = Rx_3 + Rx_4 \)

\(\displaystyle \Longrightarrow \)

$x_1x_3 + x_2x_4 = (x_1f + x_2g)(x_3h + x_4k),\ f,g,h,k \in K[x_1,x_2,x_3,x_4]$ ... ... (1)

(Is this my error ... I am assuming that (1) is simply coming from \(\displaystyle x_1x_3 + x_2x_4 \in IJ \) and the definitions of I and J.)

So then we show ... ? ... that no such f, g, h , k exist ?

So on my assumption about (1) this (ridiculously) seems to show that \(\displaystyle x_1x_3 + x_2x_4 \notin IJ \) ? and not that
\(\displaystyle x_1x_3 + x_2x_4 \notin \{fg \ | \ f \in I, g \in J \} \)

If the equation (1) was a condition for \(\displaystyle x_1x_3 + x_2x_4 \) to belong to \(\displaystyle \{fg \ | \ f \in I, g \in J \} \) then I could understand the proof.

Can you help further?

Many apologies for being slow here.

Peter
 
  • #8
No...condition (1) is indeed what we need for $x_1x_3 + x_2x_4$ to belong to the "product set" (not the "product ideal").

It should be self-evident that $x_1x_3 + x_2x_4 \in IJ$, since each term of the sum is. Since I later show it is not in the product set, the two sets cannot be equal.
 
  • #9
Deveno said:
No...condition (1) is indeed what we need for $x_1x_3 + x_2x_4$ to belong to the "product set" (not the "product ideal").

It should be self-evident that $x_1x_3 + x_2x_4 \in IJ$, since each term of the sum is. Since I later show it is not in the product set, the two sets cannot be equal.

Yes, just realized this a moment ago ... thanks for all the posts on this exercise ...

Peter
 

Related to Products of ideals of K[x1, x2, x3, x4]

1. What are "Products of ideals of K[x1, x2, x3, x4]"?

"Products of ideals of K[x1, x2, x3, x4]" refers to the multiplication of two or more ideals in the polynomial ring K[x1, x2, x3, x4]. An ideal is a subset of a ring (in this case, the polynomial ring) that satisfies certain properties, such as closure under addition and multiplication.

2. Why are products of ideals important in mathematics?

Products of ideals are important because they allow us to study and manipulate polynomials in a more efficient way. They also play a crucial role in algebraic geometry, where ideals are used to describe geometric objects.

3. How do you compute the product of two ideals in K[x1, x2, x3, x4]?

To compute the product of two ideals in K[x1, x2, x3, x4], you can use the distributive property of multiplication over addition. This means multiplying each element in the first ideal with each element in the second ideal, and then combining like terms.

4. Can products of ideals be simplified?

Yes, products of ideals can be simplified using various techniques, such as factoring, reducing to a minimal generating set, or finding a Gröbner basis. These techniques allow for a more concise representation of the product and can make calculations easier.

5. How are products of ideals related to the quotient ring?

The quotient ring is a ring that is formed by dividing a larger ring by an ideal. Products of ideals can be used to describe the structure of the quotient ring. In fact, the quotient ring is isomorphic to the product of the quotient rings of each individual ideal in the product of ideals.

Similar threads

  • Linear and Abstract Algebra
Replies
8
Views
1K
  • Linear and Abstract Algebra
Replies
1
Views
1K
  • Special and General Relativity
Replies
13
Views
1K
  • Calculus and Beyond Homework Help
Replies
5
Views
2K
  • Advanced Physics Homework Help
Replies
5
Views
2K
  • Special and General Relativity
4
Replies
108
Views
7K
  • Linear and Abstract Algebra
Replies
2
Views
1K
Replies
2
Views
1K
  • Advanced Physics Homework Help
Replies
1
Views
1K
Back
Top